Polynomial finite fields; ElGamal decryption

Click For Summary
The discussion focuses on decrypting an ElGamal encrypted message using polynomial finite fields. The key points include the public and private keys, as well as the encryption formula, where the challenge lies in calculating r^-a in F_q. The example provided illustrates the decryption process using Fermat's Little Theorem to simplify calculations in the finite field F_23. Participants clarify that 21^-6 can be expressed as 21^16, and further explain how to derive this equivalence. Understanding these concepts is crucial for successfully decrypting messages in ElGamal encryption.
LANS
Messages
24
Reaction score
0

Homework Statement


Given some ElGamal private key, and an encrypted message, decrypt it.


Homework Equations


Public key (F_q, g, b)
Private key a such that b=g^a

Message m encrypted so that r=g^k, t=mb^k

Decrypt: tr^-a = m

The Attempt at a Solution



My problem is finding r^-a. I don't quite get polynomial finite fields, and I can't figure out how to get x such that r^x = r^-a in F_q.



I have an example from the textbook:
Public key (F_23, 7, 4), private key a=6, encrypted message r=21, t=11

The textbook then does 11*21^-6 = 11*21^16 = 11*2^16 = 11*9 = 7

I understand why 11*2^16 = 7 in F_23, but I don't get how 21^-6 = 21^16 and how 21^16 = 2^16.

Any help is appreciated.
 
Physics news on Phys.org
LANS said:
I understand why 11*2^16 = 7 in F_23, but I don't get how 21^-6 = 21^16 and how 21^16 = 2^16.

Since 23 is a prime number, the field F23 is essentially just the set {0,1,2,...,22} with addition and multiplication carried out modulo 23.

Do you know Fermat's Little Theorem? It says that for any prime p and and any integer a, we have ap-1 ≡ 1 (mod p), which you can use to show 21-6 = 2116 in F23.

Also, since addition in F23 is done modulo 23, you have x = 23 - x for all x ∈ F23, which gives you 2116 = 216.
 
Thanks, that helps.

Yes, I do know Fermat's little theorem, I feel silly now for not thinking of it.
 
Question: A clock's minute hand has length 4 and its hour hand has length 3. What is the distance between the tips at the moment when it is increasing most rapidly?(Putnam Exam Question) Answer: Making assumption that both the hands moves at constant angular velocities, the answer is ## \sqrt{7} .## But don't you think this assumption is somewhat doubtful and wrong?

Similar threads

Replies
14
Views
2K
  • · Replies 1 ·
Replies
1
Views
2K
  • · Replies 1 ·
Replies
1
Views
2K
  • · Replies 1 ·
Replies
1
Views
2K
  • · Replies 1 ·
Replies
1
Views
2K
  • · Replies 5 ·
Replies
5
Views
3K
  • · Replies 6 ·
Replies
6
Views
3K
  • · Replies 28 ·
Replies
28
Views
5K
  • · Replies 4 ·
Replies
4
Views
2K
  • · Replies 5 ·
Replies
5
Views
2K